2
$\begingroup$

Let $\nu$ be a probability measure equivalent to $\mathbf{1}_{\mathbb{R}_+}(y) \, \lambda(dy)$. Let $\pi$ be a probability measure on $\mathbb{R}^2$ of second marginal $\nu$, such that $\nu(dy)$-a.e., $$ \pi_{Y = y}(dx) = \frac{\mathbf{1}_{A_y}(x)}{\lambda(A_y)} \, dx, $$ with $A_y \;=\; \bigl[\{y\} - 2^{-\lfloor y \rfloor},\;\{y\} + 2^{-\lfloor y \rfloor}\bigr] \cap [0,1]$, where $\{y\}$ denotes the fractional part of $y$ and $\lfloor y \rfloor$ denotes the floor of $y$. Since $\mu$ is absolutely continuous with respect to Lebesgue measure $\lambda$ on $[0,1]$, there exists a density $m : [0,1] \to \mathbb{R}_+$ such that $$ \mu(dx) \;=\; m(x)\,\lambda(dx), $$ the conditional law of $Y$ given $X=x$ admits the following expression: $$ \pi_{X=x}(dy) \;=\; \frac{\mathbf{1}_{A_y}(x)/\lambda(A_y)}{m(x)}\,\nu(dy), \qquad \text{for $\mu$-a.e. } x\in[0,1]. $$

The operator $$ T : L^1(\nu) \to L^1(\mu), $$ \begin{gather*} Tg(x) = \int_{y \in \mathcal{Y}} g(y) \, \pi_{X=x}(dy) \\ = \frac{1}{m(x)} \int_{\mathcal{Y}} g(y)\,\frac{\mathbf{1}_{\bigcup_{n \in \mathbb{N}} \bigl([n + x - 2^{-n},\, n + x + 2^{-n}] \cap [n, n+1)\bigr)}(y)}{\lambda(A_y)}\,\nu(dy) =\mathbb{E}[g(Y) | X = x]. \label{eq:def_T_operator} \end{gather*} is surjective.

I show the surjectivity by showing that the operator adjoint $T^*$ defined as $$T^* : L^\infty(\mu) \to L^\infty(\nu)$$ $$T^*f(y) = \int_{x \in \mathbb{R}} f(x) \, \pi_{Y=y} (dx)= \mathbb{E}[f(X)|Y=y]$$ is lower bounded (proof below).

My question: I wonder if we can be more explicit and for every function $f$ in $L_1(\mu)$, construct a function $g$ in $L_1(\nu)$, such that $Tg=f$. This amount to solving explicitly

$$ \mathbf{1}_{[0,1]}\lambda(dx)\text{-a.e., }\quad f(x) = \int_{y \in \mathbb{R}^+} g(y)\, \mathbf{1}_{\bigcup_{n \in \mathbb{N}} [n + x - 2^{-n},\, n + x + 2^{-n}] \cap [n, n+1)}(y)\, \lambda(dy). $$

Proof of surjectivity Let $\delta\in(\frac{1}{2},1)$. We are going to check that $ \nu\bigl(\{\,y \in \mathbb{R}_+:\,\pi_{Y=y}(A)>\delta\}\bigr)>0 $ for all $A\in \mathcal{B}(\mathbb{R})$ such that $\mu(A)>0$. Let $A\in\mathcal{B}(\mathbb{R})$ be such that $\mu(A)>0$. Without loss of generality, we may assume that $A \in \mathcal{B}([0,1])$, since $\operatorname{supp}(\mu) \subset [0,1]$. By the Lebesgue differentiation theorem, we have $$ \mathbf{1}_A(x)\, \lambda(dx) \text{-a.e., } \quad \frac{\lambda(A \cap [x-\varepsilon, x])}{\lambda([x-\varepsilon, x])} \xrightarrow[\varepsilon \to 0^+]{} 1,\quad \frac{\lambda(A \cap [x, x+\varepsilon))}{\lambda([x, x+\varepsilon))} \xrightarrow[\varepsilon \to 0^+]{} 1. $$ Hence, there exist $x\in(0,1)$ and $\varepsilon>0$ such that $[x-\varepsilon,x+\varepsilon]\subset[0,1]$ and $$ \forall \varepsilon' \in (0, \varepsilon), \quad \frac{\lambda\bigl(A\cap[x-\varepsilon',x]\bigr)}{\varepsilon'} >\delta, \quad \frac{\lambda\bigl(A\cap[x,x+\varepsilon']\bigr)}{\varepsilon'} >\delta. $$ In particular, for every interval $I$ of positive length, such that $I \subset [x-\varepsilon, x+\varepsilon]$ and $x \in I$, we have $$ \frac{\lambda\bigl(A\cap I\bigr)}{\lambda(I)} > \delta. $$ Let $n\in\mathbb{N}$ and $y\in[n,n+1)$. We observe that $$ y \in \left[ n + x - \min\left(2^{-n},\, \varepsilon - 2^{-n} \right),\ n + x + \min\left(2^{-n},\, \varepsilon - 2^{-n} \right) \right] \quad \text{and} \quad 2^{-n} \leq \varepsilon $$ implies that $$ A_y \subset [x - \varepsilon,\, x + \varepsilon] \quad \text{and} \quad x \in A_y. $$ Thus for $n\geq \lceil-\log_2\varepsilon\rceil +1$, we have $2^{-n}\le \frac{e^{-\lceil-\log_2\varepsilon\rceil\log 2}}2\le \frac{\varepsilon}2$, so that $$ [n+x-2^{-n},n+x+2^{-n}] \subset \{\,y\in[n,n+1):A_y\subset[x-\varepsilon,x+\varepsilon] \text{ and } x\in A_y \}. $$ Therefore, by hypothesis on $\nu$, we have $ \nu\left(\left\{\, y \in [n, n+1) : A_y \subset [x - \varepsilon, x + \varepsilon]\text{ and } x\in A_y \,\right\}\right)> 0. $ It follows that $$ \nu\left(\left\{\,y \in \mathbb{R}_+:\,\pi_{Y=y}(A) > \delta\,\right\}\right) = \nu\left(\left\{\,y \in \mathbb{R}_+:\,\frac{\lambda(A \cap A_y)}{\lambda(A_y)} > \delta\,\right\}\right) \ge \nu\left(\left\{\,y \in \mathbb{R}_+:\, A_y \subset [x - \varepsilon, x + \varepsilon] \text{ and } x\in A_y \right\}\right) > 0.$$ Let $ f^* $ be a simple function in $ L^\infty(\mu) $.
There exists a finite index set $ I $, a collection of disjoint Borel sets $ (A_i)_{i \in I} $ with $ \mu(A_i) > 0 $,
and a vector $ (f^*_i)_{i \in I}\in \mathbb{R}^I$ such that $$ \forall x \in \mathbb{R},\text{ }f^*(x) = \sum_{i \in I} f^*_i\, \mathbf{1}_{A_i}(x). $$ Let $ i_0 \in I $ such that $\max_{i \in I} \lvert f_i^* \rvert = \lvert f^*_{i_0} \rvert = \lVert f^*\rVert_{L^\infty(\mu)}$. We have $\nu(dy)$-a.e., \begin{align*} \left| T^*f^*(y) \right| &= \left| \sum_{i \in I} f^*_i \, \pi_{Y=y}(A_i) \right| \\ &\geq \left| f^*_{i_0} \right| \pi_{Y=y}(A_{i_0}) - \sum_{i \in I \setminus \{i_0\}} \left| f^*_{i_0} \right| \pi_{Y=y}(A_i) \\ &= \left( 2\, \pi_{Y=y}(A_{i_0}) - 1 \right) \left| f^*_{i_0} \right|. \end{align*} Thus $T^*$ is lower-bounded, so that $T$ is surjective.

$\endgroup$
2
  • 1
    $\begingroup$ This (nice) question would maybe get more interest if you state it without the probabilistic fluff as a real analysis type question (explicitly writing out $Tg$, maybe even without the normalising constant) as sum of averages over balls. $\endgroup$ Commented Nov 15 at 11:20
  • $\begingroup$ @unwissen Oh I see! I modified the post, is this better like this? $\endgroup$ Commented Nov 15 at 20:50

0

You must log in to answer this question.

Start asking to get answers

Find the answer to your question by asking.

Ask question

Explore related questions

See similar questions with these tags.